Complex Analysis: Contour Integration Question

Click For Summary
The discussion focuses on the application of the Fundamental Theorem of Contour Integration to two integrals involving the functions 1/z and the complex conjugate of z. For the integral of 1/z, it is initially argued that the theorem cannot be applied due to the lack of a global antiderivative, but later it is suggested that a suitable branch of the logarithm can be used within a defined open set that does not encircle the origin. In contrast, the integral of the conjugate function does not appear to have an antiderivative, leading to the conclusion that the theorem cannot be applied here. Both integrals yield the same result of i(7π/6), which raises questions about the reasoning behind their evaluations. The discussion highlights the nuances of contour integration and the conditions under which the Fundamental Theorem can be applied.
Pyroadept
Messages
82
Reaction score
0

Homework Statement


State, with justification, if the Fundamental Theorem of Contour Integration can be applied to the following integrals. Evaluate both integrals.

\begin{eqnarray*}
(i) \hspace{0.2cm} \int_\gamma \frac{1}{z} dz \\
(ii) \hspace{0.2cm} \int_\gamma \overline{z} dz \\
\end{eqnarray*}

where

\begin{eqnarray*}
\gamma(t) = \exp(i2\pi t), -\frac{1}{4} \leq t \leq \frac{1}{3}
\end{eqnarray*}

Homework Equations



Fundamental Theorem of Contour Integration:

\begin{eqnarray*}
\text{Suppose }f : S \rightarrow C\text{ is continuous, }\\
\gamma : \left[a:b\right] \rightarrow S\text{ is a smooth path.}\\
\text{Suppose }F : S \rightarrow C\text{ such that }F'(z) = f(z) \text{ }\forall z \in S.\\
\text{Then } \int_\gamma f dz = F(\gamma(b)) - F(\gamma(a))
\end{eqnarray*}Complex Line Integral Formula:

\begin{eqnarray*}
\text{Let }U \subset C \text{ be an open path-connected set. }\\
\text{Let }f : U \rightarrow C \text{ be a continuous function.} \\
\text{Let }\gamma : \left[a:b\right] \rightarrow S \text{ be a smooth path in U.}\\
\text{Then the complex line integral of f along }\gamma \text{ is:} \\
\int_\gamma f dz = \int_{a}^{b} f(\gamma(t))\gamma'(t)dt, \hspace{0.2cm} \text{where }\gamma'(t) = x'(t) + iy'(t)
\end{eqnarray*}

The Attempt at a Solution



Hi everyone,

Here's my attempt so far:\begin{eqnarray*}
\text{(i) }\hspace{0.2cm} \int_\gamma \frac{1}{z} dz
\end{eqnarray*}

Can't use Fundamental Theorem as 1/z has no antiderivative. Even though we can differentiate Log(z) to get 1/z, this is only defined locally, not generally, as Log(z) is itself based on the complex exponential, which is a periodic function.

Therefore, use the complex line integral formula:

\begin{eqnarray*}
\gamma(t) &=& \exp(i2\pi t) \\
\gamma'(t) &=& i2\pi \exp(i2\pi t) \\
f'(\gamma(t)) &=& \frac{1}{\exp(i2\pi t)} \\
&=& \exp(-i2\pi t)\\
\hspace{0.2cm} \int_\gamma \frac{1}{z} dz &=& \int_{-\frac{1}{4}}^{\frac{1}{3}}\exp(-i2\pi t)i2\pi\exp(i2\pi t)dt \\
&=& \int_{-\frac{1}{4}}^{\frac{1}{3}}i2\pi dt \\
&=&\left[i2\pi t\right]_\frac{-1}{4}^\frac{1}{3} \\
&=& i\frac{7\pi}{6}
\end{eqnarray*}\begin{eqnarray*}
\text{(ii) }\hspace{0.2cm} \int_\gamma \overline{z} dz
\end{eqnarray*}

Again, can't use Fundamental Theorem as z conjugate has no antiderivative. Therefore, use the complex line integral formula:

\begin{eqnarray*}
\gamma(t) &=& \exp(i2\pi t) \\
\gamma'(t) &=& i2\pi \exp(i2\pi t) \\
f(\gamma(t)) &=& \overline{\exp(i2\pi t)} \\
&=& \exp(-i2\pi t)\\
\int_\gamma \overline{z} dz &=& \int_{-\frac{1}{4}}^{\frac{1}{3}}\exp(-i2\pi t)i2\pi\exp(i2\pi t)dt \\
&=& i\frac{7\pi}{6} \\
\end{eqnarray*}

as before.


It seems strange that I get the same answer for both parts. I am reasonably happy that my answer for (ii) is correct. I think I may be wrong about the first one, particularly in my reasoning that it has no antiderivative. Also, seeing as the questions are from a chapter on contour integration, it seems strange that neither one can use the fundamental theorem.

I'd really appreciate if anyone could help point out where I may be going wrong!

Thanks
 
Physics news on Phys.org
Pyroadept said:
It seems strange that I get the same answer for both parts.
I don't think that's a concern, because the contour contains only numbers with unit modulus, and for such numbers the reciprocal is the same as the conjugate, ie ##\tfrac{1}{z}=\bar z##.

Pyroadept said:
I think I may be wrong about the first one, particularly in my reasoning that it has no antiderivative.
I think you can use the Fundamental Theorem at least for the ##1/z## case, because all that's necessary is for the antiderivative to exist on an open set containing the contour ##D\equiv Image\ \gamma##. It doesn't have to exist everywhere on ##\mathbb{C}##. So we can define our open set ##S## as all points no more than 0.001 away from a point in ##D## (an open 'ribbon' containing ##D##) then you can choose as antiderivative of ##1/z## a branch of the complex logarithm function that doesn't have a cut in ##S##, which is easy because ##S## doesn't entirely encircle the origin.

I'm not sure whether the conjugate has an antiderivative on ##S##. I can't think of one. I suspect it doesn't as, given the way the question is worded, it seems likely that the Fund Theorem will be applicable to one but not the other of the two cases. Unfortunately, my Complex Analysis is too rusty to produce a proof that the conjugate has no antiderivative on ##S##.
 
Question: A clock's minute hand has length 4 and its hour hand has length 3. What is the distance between the tips at the moment when it is increasing most rapidly?(Putnam Exam Question) Answer: Making assumption that both the hands moves at constant angular velocities, the answer is ## \sqrt{7} .## But don't you think this assumption is somewhat doubtful and wrong?

Similar threads

Replies
1
Views
1K
Replies
9
Views
2K
  • · Replies 9 ·
Replies
9
Views
2K
  • · Replies 8 ·
Replies
8
Views
2K
  • · Replies 1 ·
Replies
1
Views
2K
Replies
13
Views
2K
Replies
32
Views
3K
  • · Replies 1 ·
Replies
1
Views
2K
  • · Replies 2 ·
Replies
2
Views
3K
Replies
23
Views
2K